Add (1.3t3 + 0.4t2 – 24t) + (8 – 18t + 0.6t2) For each term in the second polynomial, enter the letter showing where that term should be placed to add the polynomials vertically.

Answers

Answer 1

Answer:

[tex](1.3t^3 + 0.4t^2 - 24t) + (8 - 18t + 0.6t^2) = 1.3t^3 + t^2 -42t + 8[/tex]

Step-by-step explanation:

Given

[tex](1.3t^3 + 0.4t^2 - 24t) + (8 - 18t + 0.6t^2)[/tex]

Required

Solve

We have:

[tex](1.3t^3 + 0.4t^2 - 24t) + (8 - 18t + 0.6t^2)[/tex]

Collect like terms

[tex]1.3t^3 + 0.6t^2 + 0.4t^2 - 24t- 18t + 8[/tex]

[tex]1.3t^3 + t^2 -42t + 8[/tex]

So:

[tex](1.3t^3 + 0.4t^2 - 24t) + (8 - 18t + 0.6t^2) = 1.3t^3 + t^2 -42t + 8[/tex]

Answer 2

Answer:

look at pic

Step-by-step explanation:

Add (1.3t3 + 0.4t2 24t) + (8 18t + 0.6t2) For Each Term In The Second Polynomial, Enter The Letter Showing

Related Questions

log8-log4 ÷ log4-log2=





Answers

The answer is log(4)-1

If
5
3 cosα = in the first quadrant, what does cot α
equal?

Answers

Answer:

5/4

Step-by-step explanation:

To Find :-

cot a .

Solution :-

By question ,

=> cos a = 5/3 = b/h

=> p = √ 5² - 3² = √ 25 -9 = 4

Therefore ,

=> cot a = b/p = 5/4

Which point is in the solution set of this system inequalities?

A. (0,0)

B. None of these

C. (5,1)

D. (3,7)

Answers

Answer:

B

Step-by-step explanation:

To find which ordered pairs are solutions to the inequalities we can simply plug in the x and y values of the ordered pairs into the inequalities and if the equation is true for both inequalities then the ordered pair is a solution to the inequalities.

For (0,0)

x = 0

y = 0

y > x + 5

Substitute 0 for y and x

0 > 0 + 5

Simplify right side

0 > 5

The inequality is not true as 5 is greater than 0, not less than. So immediately we can eliminate answer choice A.

For (5,1).

x = 5

y = 1

y > x + 5

Substitute 5 for x and 1 for y

1 > 5 + 5

Simplify right side

1 > 10

Again, the equation is not true as 1 is not greater than 10. This means that c cannot be the answer

For (3,7)

x = 3

y = 7

y > x + 5

Substitute 3 for x and y for 7

7 > 3 + 5

Simplify right side

7 > 8

7 is not greater than 8 meaning that (3,7) cannot be a solution to the inequalities

None of the ordered pairs created true equations hence the answer is B

Arvin has $10000 to invest. He invests part in a term deposit paying 5%/year, and the remainder in Canada savings bonds paying 3.4%/year. At the end of the year, he earned simpler interest of $413. How much did he invest at 5%/year?

Answers

Answer:

$4,562.5

Step-by-step explanation:

The amount Arvin has to invest, P = $10,000

The interest paid on the investment in the term deposit = 5%/year

The interest paid om the investment in Canada savings bonds = 3.4%/year

The amount Arvin earned at the of the year as simple interest, A = $413

Let, x, represent the amount Arvin invested in the term deposit and let, y, represent the amount he invested in Canada savings bonds, we can get the following system of equations

x + y = 10,000...(1)

0.05·x + 0.034·y = 413...(2)

Making y the subject of equation (1) and substituting the value in equation (2), we get;

From equation (1), we get, y = 10,000 - x

Plugging the above value of y in equation (2) gives;

0.05·x + 0.034 × (10,000 - x) = 413

∴ 0.05·x - 0.034·x + 340 = 413

x = (413 - 340)/(0.05 - 0.034) = 4,562.5

Therefore, the amount Arvin invested in the term deposit at 5%, x = $4,562.5

(y = 10,000 - x

∴ y = 10,000 - 4,562.5 = 5,437.5

The amount Arvin invested in Canada savings bonds, y = $5,437.5.)

If f(x) = 2x2 + 1, what is f(x) when x = 3?

Answers

Answer:

18

Step-by-step explanation:

f(x)=2x(2+1)

f(3)=2(3)(2+1)

    =6(2+1)

    =12+6

    =18


[tex]solve : - \\ \\ (4 {}^{2} + 5 {}^{2} ) = {?}[/tex]

Answers

Step-by-step explanation:

4² = 16

5² = 25

16+25 = 41

41 is the answer.

Hope it helps! :)

Answer:

[tex]( {4}^{2} + {5}^{2} ) \\ (16 + 25) \\ = 41[/tex]

A ABC has vertices A (-7, -13), B (12, -8), and C(-17, 19). Which of the following represents the
reflection of AABC across the line y = x and its rotation of 90° about the origin?

Answers

Answer:

brainliest and 20 points! Which best explains whether or not ΔABC ≅ ΔLMN? The figures are congruent because a 270° rotation about the origin and then a reflection over the x-axis will map ΔABC onto ΔLMN. The figures are congruent because a 90 rotation about the origin and then a reflection over the x-axis will map ΔABC onto ΔLMN. The figures are not congruent because point B corresponds with point N and point C corresponds with point M. The figures are not congruent because there is no rigid transformation or combination of rigid transformations that will map ΔABC onto ΔLMN.

Step-by-step explanation:

it's is the answr

The measure of an angle is eight times the measure of its supplementary angle. What is the measure of each angle?

Answers

Answer:

160°

Step-by-step explanation:

Let one angle be x

Let second angle be 8x

Sum of supplementary angles = 180°

x+ 8x = 180°

9x = 180

x = 180/9 = 20

First angle= x= 20°

Second angle = 8x = 8*20 = 160°

The sum of three consecutive odd numbers is 63. What are the numbers?​

Answers

19,21,23. Adding all of these consecutive odd numbers is equal to 63

Answer:

19, 21 and 23

Step-by-step explanation:

→ Make an algebraic expression for the 3 consecutive numbers

2x + 1, 2x + 3 and 2x + 5

→ Add the expressions together and make it equal to 63

2x + 1 + 2x + 3 + 2x + 5 = 63

→ Simplify

6x + 9 = 63

→ Minus 9 from both sides

6x = 54

→ Divide both sides by 6

x = 9

→ Resubstitute x = 9 into the 3 expressions

x = 9 into 2x + 1 is 19

x = 9 into 2x + 3 is 21

x = 9 into 2x + 5 is 23

I would raise it to 1grand but I’m very broke

Answers

Answer:

ITS A LOL

Step-by-step explanation:

what is the value of b in this equation?

A. -20
B. -6
C. 6
D. 20

Answers

Answer:

Step-by-step explanation:

a- 12

b- 2

c- 11

and the next part is

x- 6

y- 18

Answer:

B

Step-by-step explanation:

Using the rules of exponents

[tex](a^m)^{n}[/tex] = [tex]a^{mn}[/tex]

[tex]a^{-m}[/tex] = [tex]\frac{1}{m}[/tex]

Then

[tex](y^b)^{4}[/tex] = [tex]\frac{1}{y^{24} }[/tex]

[tex]y^{4b}[/tex] = [tex]y^{-24}[/tex]

Equate the exponents

4b = - 24 ( divide both sides by 4 )

b = - 6 → C

What is 6.273 rounded to the nearest thousandths?

Answers

Answer:

6.270

Step-by-step explanation:

3 is below 5 so you just turn it into a 0. If it was 5 or above you would just add a number to the 7.

Does anyone know the answer to this question?

Answers

The answer is A; g(x) = |x+4| - 2

Need help ASAP !!!when creating a question for a survey , the way a question is written:

Answers

Answer:

The third one i think

Step-by-step explanation:

Since a survey shouldn't be biased or anything.

In In 5x + In In (x - 1) = 2

Answers

Answer:

exact form: x=-1/2

decimal form: x=-0.5

Find the value of y

Help please

Answers

Answer:

6

Step-by-step explanation:

Set your formula up as

15 = 2y+3

15 - 3 = 2y

12 = 2y

12 / 2 = y

6 = y

find the volume of the rectangular prism. plz answer this lol

Answers

Answer:

.....how when the dimensions are not even clear lol

Answer:

48 cm³

Step-by-step explanation:

the volume of a rectangular prism= length × breadth × height

= 8× 3 × 2

= 48 cm³

Find the measure of the missing angle using the exterior angle sum theorm.

Answers

Answer:

85°

Step-by-step explanation:

The exterior angle of a triangle is=sum of the opposite interior angles

So

? °=45°+40°


Find (a) the mean, (b) the median, and
(c) the mode of the number of verses in
the first eight chapters of Proverbs.
Proverbs 1 23 4 5 6 7 8
Verses 33 22352723352736
(a)
(b)
(c)

Answers

Answer:

Mean = 33+ 22+35+27+23+35+27+36 = 238/8 = 29.75

Median = 27+23 = 50/2 = 25

Mode = 35

Range = 36 - 22 = 14

Step-by-step explanation:

Stay Blessed

help help help pls :)

Answers

Answer:

[tex]opposite\approx 70.02[/tex]

Step-by-step explanation:

The triangle in the given problem is a right triangle, as the tower forms a right angle with the ground. This means that one can use the right angle trigonometric ratios to solve this problem. The right angle trigonometric ratios are as follows;

[tex]sin(\theta)=\frac{opposite}{hypotenuse}\\\\cos(\theta)=\frac{adjacent}{hypotenuse}\\\\tan(\theta)=\frac{opposite}{adjacent}[/tex]

Please note that the names ([tex]opposite[/tex]) and ([tex]adjacent[/tex]) are subjective and change depending on the angle one uses in the ratio. However the name ([tex]hypotenuse[/tex]) refers to the side opposite the right angle, and thus it doesn't change depending on the reference angle.

In this problem, one is given an angle with the measure of (35) degrees, and the length of the side adjacent to this angle. One is asked to find the length of the side opposite the (35) degree angle. To achieve this, one can use the tangent ([tex]tan[/tex]) ratio.

[tex]tan(\theta)=\frac{opposite}{adjacent}[/tex]

Substitute,

[tex]tan(35)=\frac{opposite}{100}[/tex]

Inverse operations,

[tex]tan(35)=\frac{opposite}{100}[/tex]

[tex]100(tan(35))=opposite[/tex]

Simplify,

[tex]100(tan(35))=opposite[/tex]

[tex]70.02\approx opposite[/tex]

if u help me i will give you brainliest <3

Answers

Answer:

a) 6

b) 12:35 ( not so sure about this one)

Step-by-step explanation:

a)

distance = speed * time

ruths distance = 4* 1.5 (because it takes her 1h 30 mins)

distance = 6miles    

b)  1/4 here is quarter so, 15mins

10:30 and then add 50 mins to that.

     she leaves at 11:20 and takes 1h and 15 mins ( 75mins) to walk home

     so 11:20 + 1h 15min

     = 12:35

hope this helps :)

Answer:

a) 6 miles

b) 12:35 pm

Step-by-step explanation:

a) 9 am to 10:30 am = 1 hour and 30 mins = 1.5 hours

4mph x 1.5 = 6miles

b) 10:30 am + 50 mins = 11:20 am

1 hour = 60 mins

60/4 = 15 mins

1 1/4 = 1 hour 15 mins

11:20 am + 1 hour 15 mins = 12:35 pm

a 10 foot ladder rests against a vertical wall if the bottom of the ladder slides away from the wall at a speed of 2 ft/s how fast is the angle betwween the top of the ladder and the wall changing when that angle is

Answers

Answer:

d∅/dt = √2/5 Rad/sec

Step-by-step explanation:

According to the Question,

Given That, a 10-foot ladder rests against a vertical wall if the bottom of the ladder slides away from the wall at a speed of 2 ft/s how fast is the angle between the top of the ladder and the wall changing when that angle is π/4.

Solution,

Let x be the Distance between the base of the wall and the bottom of the ladder.

and let ∅ be the angle between the top of the ladder and the wall.

Then, Sin∅ =x/10  so, x=sin∅ *10

Differentiating with respect to time t we get,

dx/dt = 10 * cos∅ * d∅ /dt

We have given that dx/dt = 2 ft/s and ∅ =π/4

Now, Put these value we get

2 = 10 *(cos(π/4))* d∅/dt

2 = 10/√2 * d∅/dt

d∅/dt = √2/5 Rad/sec

a^2×c^2/c^2×d^2+bc/ad reduce the algebraic ​

Answers

Answer:

[tex]\frac{a^2*c^2}{c^2*d^2}+\frac{bc}{ad}= \frac{a^3 + bcd}{ad^2}[/tex]

Step-by-step explanation:

Given

[tex]\frac{a^2*c^2}{c^2*d^2}+\frac{bc}{ad}[/tex]

Required

Simplify

We have:

[tex]\frac{a^2*c^2}{c^2*d^2}+\frac{bc}{ad}[/tex]

Cancel out [tex]c^2[/tex]

[tex]\frac{a^2*c^2}{c^2*d^2}+\frac{bc}{ad}= \frac{a^2}{d^2}+\frac{bc}{ad}[/tex]

Take LCM

[tex]\frac{a^2*c^2}{c^2*d^2}+\frac{bc}{ad}= \frac{a^3 + bcd}{ad^2}[/tex]

plz help me with this math and also explain

Answers

Step-by-step explanation:

[1]

SI = $250Rate (R) = 12[tex] \sf \dfrac{1}{2}[/tex] %Time (t) = 4 years

[tex]\longrightarrow \tt { SI = \dfrac{PRT}{100} } \\ [/tex]

[tex]\longrightarrow \tt { 250 = \dfrac{P \times 12\cfrac{1}{2} \times 4}{100} } \\ [/tex]

[tex]\longrightarrow \tt { 250 = \dfrac{P \times \cfrac{25}{2} \times 4}{100} } \\ [/tex]

[tex]\longrightarrow \tt { 250 = \dfrac{P \times 25 \times 2}{100} } \\ [/tex]

[tex]\longrightarrow \tt { 250 = \dfrac{P \times 50}{100} } \\ [/tex]

[tex]\longrightarrow \tt { 250 \times 100 = P \times 50} \\ [/tex]

[tex]\longrightarrow \tt { 25000 = P \times 50} \\ [/tex]

[tex]\longrightarrow \tt { \dfrac{25000}{50} = P } \\ [/tex]

[tex]\longrightarrow \underline{\boxed{ \green{ \tt { \$ \; 500 = P }}}} \\ [/tex]

Therefore principal is $500.

__________________

[2]

2/7 of the balls are red.3/5 of the balls are blue.Rest are yellow.Number of yellow balls = 36

Let the total number of balls be x.

→ Red balls + Blue balls + Yellow balls = Total number of balls

[tex]\longrightarrow \tt{ \dfrac{2}{7}x + \dfrac{3}{5}x + 36 = x} \\ [/tex]

[tex]\longrightarrow \tt{ \dfrac{10x + 21x + 1260}{35} = x} \\ [/tex]

[tex]\longrightarrow \tt{ \dfrac{31x + 1260}{35} = x} \\ [/tex]

[tex]\longrightarrow \tt{ 31x + 1260= 35x} \\ [/tex]

[tex]\longrightarrow \tt{ 1260= 35x-31x} \\ [/tex]

[tex]\longrightarrow \tt{ 1260= 4x} \\ [/tex]

[tex]\longrightarrow \tt{ \dfrac{1260 }{4}= x} \\ [/tex]

[tex]\longrightarrow \underline{\boxed{ \tt { 315 = x }}} \\ [/tex]

Total number of balls is 315.

A/Q,

3/5 of the balls are blue.

[tex]\longrightarrow \tt{ Balls_{(Blue)} =\dfrac{3 }{5}x} \\ [/tex]

[tex]\longrightarrow \tt{ Balls_{(Blue)} =\dfrac{3 }{5}(315)} \\ [/tex]

[tex]\longrightarrow \tt{ Balls_{(Blue)} = 3(63)} \\ [/tex]

[tex]\longrightarrow \underline{\boxed{ \green {\tt { Balls_{(Blue)} = 189 }}}} \\ [/tex]

The polygons are similar, but not necessarily drawn to scale. Find the value of x. PLEASE HELPPPP

Answers

Answer:

x = 27.5.

Step-by-step explanation:

There are given numbers on each side. If the figures are similar, then they have a set ratio for each value.

So, 55:8 and x:4. If you want to, you can flip it, so that it is 8:55 and 4:x.

With that in mind, it is easy to see what the ratio is. Because 4 is half of 8, x is half of 55. 55 divided by 2 is 27.5.

Therefore, x = 27.5.

In which section of the number line is 32−−√?

Answers

where's the number line?

maybe u can attach it at the comments:)

Answer:

Section B

Step-by-step explanation:

The equation d = m/v can be used to calculate the density,d, of an object with mass,m, and volume,V. Which is an equivalent equation solver for V?

Answers

Answer:

v = m/d

Step-by-step explanation:

since

[tex]d = \frac{m}{v} \: \\ \\ v = \frac{m}{d} [/tex]

For the function G defined by G(x)=5x+3, find G(r+5)

Answers

Given function:

g(x) = 5x + 3

Find

g(r+5)

Substitute x with r = 5:

g(r + 5) = 5(r + 5) + 3 = 5r + 25 + 3 = 5r + 28

Answer:

G ( r + 5 ) = 5r + 28

Step-by-step explanation:

Given ;

G ( x ) = 5x + 3

To Find :-

G ( r + 5 )

Solution :-

plug r + 5 as x in the function.

G ( r + 5 ) = 5 ( r + 5 ) + 3

distribute 5

G ( r + 5 ) = 5r + 25 + 3

combine like terms

G ( r + 5 ) = 5r + 28

solve the simultaneous equation: x-y=2
xy=36​

Answers

Answer:

Y= –7.08, 5.08

Step-by-step explanation:

hope ya ready bro.

X=36/Y

replace 36/y instead of X

36/Y–Y=2===> 36–Y²=2Y===> Y²+2Y–36=0

Y1= –1+√37≈ –7.08

Y2= –1–√37≈ 5.08

A lending library has a fixed charge for the first three days and an additional charge for each day thereafter. Saritha paid ₹27 for a book kept for seven days, while Susy paid ₹21 for the book she kept for five days. Find the fixed charge and the charge for each extra day.​​

Answers

Answer:

Ji Gud afternoon.

I'm fine

what about you madam.

Hope u r fine and stay safe and healthy

Other Questions
Calculate the difference and enter it below. -5 - (-10) Read these sentences from the excerpt:I would sometimes say to them, I wished I could be as free as they would be when they got to be men. "You will be free as soon as you are twenty-one, but I am a slave for life! Have not I as good a right to be free as you have?"How does the use of a rhetorical question advance the author's purpose of showing that all people deserve freedom?It demonstrates that Douglass is questioning his beliefs.It shows the reader that Douglass is unsure of his statements.It indicates that Douglass believes he deserves freedom.It reminds the reader that some questions are unanswerable. What is the cause of acid precipitation?1) ozone2) nitrogenous oxides3) ammonia4) chlorofluorocarbons (CFCs) Team members Corinne, Kevin, and Tomas decide to share the costof 2 motor controllers and 4 wheels equally. How much does eachmember need to contribute?Please help(write step by step and upload pic or do it here please I appreciate it so much ! its not 12 fyi oh btw question is in the picture btw this is middle school not high school A construct that enables you to create custom types by grouping together variables of other types is called a what What is the next three-term of the geometric sequence? 60, 30, 15...? Calculate the dose of acetaminophen for a 3-year-old child with the body surface area of 0.30 m2. The adult dose of the drug is 500 mg. Record your answer using a whole number. _____mg Find a unit vector u u in R 2 R2 such that u u is perpendicular to v . v. How many such vectors are there The nurse is trying to determine the ability of the client with myocardial infarction (MI) to manage independently at home after discharge. Which statement by the client is the strongest indicator of the potential for difficulty after discharge?1. I need to start exercising more to improve my health.2. I will be sure to keep my appointment with the cardiologist.3. I don't have anyone to help me with doing heavy housework at home.4. I think I have a good understanding of what all my medications are for. What is the quotient of Morrison is 11 years older than his wife Susana. Susana who is 11 times as old as her son .if her son attains age of 9 years after 5 years. What is the present age of Morrison. Un alambre de plstico, aislante y recto mide 10 cm de longitud y tiene una densidad de carga de +150 nC/m, distribuidos de manera uniforme por toda su longitud. Se encuentra sobre una mesa horizontal. A) Encuentre la magnitud y la direccin del campo elctrico que produce este alambre en un punto que est 8 cm directamente arriba de su punto medio. B) Si el alambre ahora se dobla para formar un crculo que se coloca aplanado sobre la mesa, calcule la magnitud y la direccin del campo elctrico que produce en un punto que se encuentra 6 cm directamente arriba de su centro. Gii phng trnh: Sin^4 x - cos^4 x =1/2 on which surfaces did you move the box easily? Suppose that you are headed toward a plateau 37 meters high. If the angle of elevation to the top of the plateau is , how far are you from the base of the plateau? A sector fund specializing in commercial bank stocks had average daily assets of $3.7 billion during the year. This fund sold $1.58 billion worth of stock during the year, and its turnover ratio was .42. How much stock did this mutual fund purchase during the year please pass the salt is an example of what sentence Please help asap!!! I'm in hurry please help I cant find the second one.